- Fri Jan 20, 2017 12:00 am
#47139
Complete Question Explanation
(The complete setup for this game can be found here: lsat/viewtopic.php?t=2909)
The correct answer choice is (B)
After two questions on the easier side, the test makers dramatically increase the difficulty with this question. The question stem first specifies that two of the windows are made with exactly two colors, and then asks for the possible color combination of one of those windows.
From our knowledge of the powerful fourth rule, answer choices (A) and (D) can immediately be eliminated because neither contains O or P. This leaves three answer choices, and the savvy test taker may realize that with two windows limited to just two colors and the second rule establishing that R must appear in two windows, that answer choice (B) is likely to be correct because it is the only remaining answer choice that contains R. However, let’s turn to the templates to determine which answer choice is correct.
Template #2 is not possible because adding the two R’s would create at least two windows with at least three colors, a violation of the condition in the question stem. Thus, Template #1 must be the applicable template for this question. When the two R’s are added to the template, one must go with the O/P option, and the other must go with either the PY or PG blocks:
As O and R is one possible complete color combination, answer choice (B) is proven correct.
(The complete setup for this game can be found here: lsat/viewtopic.php?t=2909)
The correct answer choice is (B)
After two questions on the easier side, the test makers dramatically increase the difficulty with this question. The question stem first specifies that two of the windows are made with exactly two colors, and then asks for the possible color combination of one of those windows.
From our knowledge of the powerful fourth rule, answer choices (A) and (D) can immediately be eliminated because neither contains O or P. This leaves three answer choices, and the savvy test taker may realize that with two windows limited to just two colors and the second rule establishing that R must appear in two windows, that answer choice (B) is likely to be correct because it is the only remaining answer choice that contains R. However, let’s turn to the templates to determine which answer choice is correct.
Template #2 is not possible because adding the two R’s would create at least two windows with at least three colors, a violation of the condition in the question stem. Thus, Template #1 must be the applicable template for this question. When the two R’s are added to the template, one must go with the O/P option, and the other must go with either the PY or PG blocks:
As O and R is one possible complete color combination, answer choice (B) is proven correct.
You do not have the required permissions to view the files attached to this post.